find the equation for the line that passes through the point (4,-3), and that is perpendicular to the line with the equation y= -3/4x-3​

Answers

Answer 1

Answer:

hi

Step-by-step explanation:

Answer

Slope between two given points (1,3) and (2,7) is

m=  

2−1

7−3

​  

=4

Then perpendicular slope is m  

1

​  

=  

4

−1

​  

 

The line equation with this slope is given by

y=  

4

−1

​  

x+c.......(1)

Now, the above line passes through the point (−4,−3)

⇒−3=  

4

−1

​  

×(−4)+c

⇒c=−4

Therefore required line is 4y+x+16=0 (Substitute  

c  

 in (1) and simplify)


Related Questions

PLEASE HELP ME!!! WILL GIVE BRAILIEST!!!!!!
Find the Product - Show all your work.

2.37 X 1.6

Answers

2.37 times 1.6=3.792


0.5 times 2.37=1.185
0.1 times2.37= 0.237

2.37+1.185+0.237=3.792

Hello-

Here is the way that I learned to multiply decimals. First, remove the decimal points.

2.37 X 1.6

237 X 16

Then you multiply the numbers.

237 X 16 = 3792

Now, we move the decimal point three places to the left on our answer.

3.792

3.792 is our answer.

A container built for transatlantic shipping is constructed in the shape of a right rectangular prism. Its dimensions are 2 ft by 2 ft by 12.5 ft. If the container is entirely full and, on average, its contents weigh 0.22 pounds per cubic foot, find the total weight of the contents. Round your answer to the nearest pound if necessary.

Answers

Answer:

38.81 pounds

Step-by-step explanation:

Considering the definition of right rectangular prism and its volume,  the total weight of the contents is 38.81 pounds.

Right rectangular prism

A right rectangular prism (or cuboid) is a polyhedron whose surface is formed by two equal and parallel rectangles called bases and by four lateral faces that are also parallel rectangles and equal two to two.

Volume of right rectangular prism

To calculate the volume of the rectangular prism, it is necessary to find the product of its dimensions, or of the three edges that converge at a certain vertex.

That is, to calculate the volume of a rectangular prism, multiply its 3 dimensions: length×width×height.

Volume of the container

In this case, you know that:

the dimensions of the container built are 7.5 ft by 11.5 ft by 3 ft.

the container is entirely full and, on average, its contents weigh 0.15 pounds per cubic foot.

So, the volume of the container is calculated as:

7.5 ft× 11.5 ft× 3 ft= 258.75 ft³

Then, the total weight of the contents is calculated as:

258.75 ft³×  0.15 pounds per cubic foot= 38.8125 pounds≅ 38.81 pounds

Finally, the total weight of the contents is 38.81 pounds.

A pancake recipe require 1 2/3 cups of flour to make 20 pancakes and you have 9 cups of flour.
a. How many pancakes can you make with 1 cup of flour?
b. How many pancakes can you make with 9 cups of flour?
C. Do you have enough to make 100 pancakes? Explain

Answers

Solution (a) :-

Quantity of flour needed to make 20 pancakes =

\( = 1 \frac{2}{3} \: \)

\( = \frac{3 \times 1 + 2}{3} \)

\( = \frac{5}{3} \: cups \: of \: flour\)

Then , number of pancakes that can be made from 1 cup of floor :-

\(20 \div \frac{5}{3} \)

\(20 \times \frac{3}{5} \)

\( \frac{60}{5} \)

\( = 12 \: pancakes\) .

Therefore , 12 pancakes can be made from 1 cup of floor .

Solution (b) :-

Number of pancakes that can be made with 1 cup of flour = 12 pancakes

Number of pancakes that can be made with 9 cups of flour :-

= 9 × 12

= 108 pancakes

Therefore , I can make 108 pancakes with 9 cups of flour .

Solution (c) :-

Yes , I have enough flour to make 100 pancakes . I say this because :-

With one cup of flour I can make = 12 pancakes

With 9 cups of flour I can make =

= 9 × 12

= 108 pancakes

As I can make 108 pancakes with 9 cups of flour , I can conclude that I have enough flour to make 100 pancakes .

3/8x - 17 = 10 PLEASEEEEEEeeeee

Answers

Answer: X=1/72

Step-by-step explanation:

3/8x-17=10

We move all terms to the left:

3/8x-17-(10)=0

Domain of the equation: 8x!=0

x!=0/8

x!=0

x∈R

We add all the numbers together, and all the variables

3/8x-27=0

We multiply all the terms by the denominator

-27*8x+3=0

Wy multiply elements

-216x+3=0

We move all terms containing x to the left, all other terms to the right

-216x=-3

x=-3/-216

x=1/72

Hello!☺

\(Answer:\boxed{x=72}\)

\(Explanation:\)

\(\frac{3}{8}x-17=10\)

Let's start by adding 17 to both sides:

\(\frac{3}{8}x-17+17=10+17\)

\(\frac{3}{8}x=27\)

In our second/final step, we will multiply both sides by 8/3:

\((\frac{8}{3})*(\frac{3}{8}x)=(\frac{8}{3})*27=72\\ x=72\)

Hope this helps!☺

Express as a product. 1+2cos a

Answers

Answer:

Therefore, 1 + 2cos a can be expressed as the product of 2 and (cos a/2 + 1).

1 + 2cos a = 2(cos a/2 + 1)

Step-by-step explanation:

We can use the trigonometric identity:

cos 2a = 1 - 2 sin^2 a

to rewrite 1 + 2cos a as:

1 + 2cos a = 1 + 2(1 - sin^2 a/2)

= 1 + 2 - 2(sin^2 a/2)

= 3 - 2(sin^2 a/2)

Now, using another trigonometric identity:

sin a = 2 sin(a/2) cos(a/2)

we can rewrite sin^2 a/2 as:

sin^2 a/2 = (1 - cos a)/2

Substituting this into the expression for 1 + 2cos a, we get:

1 + 2cos a = 3 - 2((1 - cos a)/2)

= 3 - (1 - cos a)

= 2 + cos a

Therefore, 1 + 2cos a can be expressed as the product of 2 and (cos a/2 + 1).

1 + 2cos a = 2(cos a/2 + 1)

Find that the radius of curvature of ^2y=x^3-a^3
at the point where the
curves cut the X-axis.

Answers

The radius of curvature of the curve \(a^{2y\)=x³-a³ at the point where the curve intersects the x-axis is 27\(a^{\frac{3}{2}\).

To find the radius of curvature of the curve \(a^{2y\)=x³-a³ at the point where the curve intersects the x-axis, we need to first find the equation of the curve and then determine the value of y and its derivative at that point.

When the curve intersects the x-axis, y=0. Therefore, we have:

a⁰ = x³ - a³

x³ = a³

x = a

Next, we need to find the derivative of y with respect to x:

dy/dx = -2x/(3a²√(x³-a³))

At the point where x=a and y=0, we have:

dy/dx = -2a/(3a²√(a³-a³)) = 0

Therefore, the radius of curvature is given by:

R = (1/|d²y/dx²|) = (1/|d/dx(dy/dx)|)

To find d/dx(dy/dx), we need to differentiate the expression for dy/dx with respect to x:

d/dx(dy/dx) = -2/(3a²(x³-a³\()^{\frac{3}{2}\)) + 4x²/(9a⁴(x³-a³\()^{\frac{1}{2}\))

At x=a, we have:

d/dx(dy/dx) = -2/(3a²(a³-a³\()^{\frac{3}{2}\)) + 4a²/(9a⁴(a³-a³\()^{\frac{1}{2}\)) = -2/27a³

Therefore, the radius of curvature is:

R = (1/|-2/27a³|) = 27\(a^{\frac{3}{2}\)

To learn more about radius of curvature click on,

https://brainly.com/question/31403088

#SPJ1

Dominics two daughters both want to take gymnastics this year. It cost 90 per month per child. How much dose dominic need to budget for a year ( 12 months) of gymnastics for both daughters

Answers

Answer:

$2,160 is what he need for both daughters for 1 year

Step-by-step explanation:

90 x 2 = 180

180 x 12 = 2,160

Dominic need to budget a total of 2160 for a year of gymnastics for both daughters.

What is Multiplication?

Multiplication is one of the basic mathematical operations which includes a number added repeatedly to times of the other number. The final answer is called the product of the numbers.

Multiplication is usually denoted as '×' sign.

a × b indicates that a is added repeatedly upto b times.

Cost for the gymnastics per month per child = 90

In order to find the cost per month for both daughters, we need to multiply 90 with 2.

Cost for the gymnastics per month for 2 daughters = 2 × 90 = 180

To find the total cost for an year, we need to multiply 180 with 12.

Total cost for the year = 180 × 12 = 2160

Hence the total budget for Dominic to send both the daughter in a year is 260.

To learn more about Multiplication, click on the link given below:

https://brainly.com/question/12017228

#SPJ5

write two and six hundreds nineteen thousandths in decimal.​

Answers

Answer:

Easy .2619

Step-by-step explanation:

hope it helped

I got “.2619” as the answer

Factorise: x^3 + x^2 + x^2y + xy + y

Plz also show me the process.

Answers

Factor out the greatest common factor from each group.
x^2 (x-1y)-y^2(x-y)

Factor the polynomial by factoring out the greatest common factor, x-1y
(x-1y)(x^2-y^2)

rewrite -1y as -y.
(x-y) (x^2-y^2)

Factor.
(x-y)(x+y)(x-y)

Combine exponents.
(x-y)^2 (x+y) (your answer)

really hoped this helped. :)




Two teams are going on a trip to play a game. There will be
2 buses to take the students. On each bus, there are 60
seats. Teachers and parents will sit in 10 of the seats on
each bus
How many seats are available for students on the buses?

No links

Two teams are going on a trip to play a game. There will be2 buses to take the students. On each bus,

Answers

-——————-
*%~.Answer .~%*
The Answer is 100
-——————-

First you would Multiply 2 and 60
You can do that by adding 60 two times.

-——————-

60. It should look like this. If you didn’t get
+ 60. 120 then your answers is incorrect, or
——— wrong. ☺️
120

-——————-

Now you multiply 10 x 2 because the teacher’s and parents will take up 10 is the seats on each bus!
To make is easier add 10 and 10 you should get 20!

-——————-

Now subtract 20 from 120.

120. You should get 100! If you didn’t get
- 20. 100 then you're answer is incorrect or
———. Wrong, Once again.
100

——-————————-

You're answer is, 100! There will be 100 seats leftover or available for the students on the busses!

~ pls Mark brainiest, or at least say thanks!

• TheTrueNezuko.

Some one plz help this is a wuick question

Some one plz help this is a wuick question

Answers

the answer is c

3.3 x10 restu power minus 3

Please help!! I’ll also give brainliest!! Thank you

Please help!! Ill also give brainliest!! Thank you

Answers

Sorry for the rough sketches.

The answer has been shaded.

Please help!! Ill also give brainliest!! Thank you

Bernie deposited $2,000 in an account earning 10% interest compounded annually. To the nearest cent, how much will he heave in 5 years?

Answers

Answer:

3000

Step-by-step explanation:

2000 x .10= 200 that's interest for one year

200 x 5 = 1000 that's interest for 5 years

1000 + 2000= 3000 total amount of money in five years

Answer:3,221.02

Step-by-step explanation:

Its factorial math pleaseHelp.

Its factorial math pleaseHelp.
Its factorial math pleaseHelp.

Answers

\(\dfrac{7!4!}{5!3!}=6\cdot7\cdot4=168\)

CAN SOMEONE PLEASE HELP me with the correct answer!!!!! I need this done please help like please

CAN SOMEONE PLEASE HELP me with the correct answer!!!!! I need this done please help like please

Answers

answer: 164 degrees

reasoning: circle = 360 degrees
360-115= 245
245-81= (164 degrees)

Question 10 (1 point)
A
33
7 in.
B
C

Question 10 (1 point)A337 in.BC

Answers

The value of AB is,

⇒ AB = 5.9

(rounded to nearest tenth)

We have to given that,

A right triangle ABC is shown.

Now, By trigonometry formula,

we get;

⇒ cos 33° = Base / Hypotenuse

Substitute all the values, we get;

⇒ cos 33° = AB / 7

⇒ 0.84 = AB / 7

⇒ AB = 0.84 × 7

⇒ AB = 5.88

⇒ AB = 5.9

(rounded to nearest tenth)

Thus, We get;

AB = 5.9

(rounded to nearest tenth)

Learn more about trigonometric ratios at:

brainly.com/question/1836193

#SPJ1

Kyle has 50 red marbles and 75 blue marbles. What is the ratio of the number of red marbles to the number of blue marbles expressed in simplest terms?

Answers

Answer: It is a 2:3 Ratio


A triangle has sides with lengths of 12 meters, 18 meters, and 19 meters. Is it a right
triangle?

Answers

Answer: yes

Step-by-step explanation:

the two sides are near the same with one smaller and i learned at sc that that makes it right

determine x in the following equation 2x - 4 = 10

Answers

Answer:

7

Step-by-step explanation:

10+4 = 14

14/2  = 7

x = 7

Find each of the following ratios. Write each ratio
in simplest form.
M-
25
O
7
N.


how do i answer number 6

Find each of the following ratios. Write each ratioin simplest form.M-25O7N. how do i answer number 6

Answers

Answer:

a.  sin(M)=7/25

b.  cos(M)=24/25

c.  tan(M)=7/24

d.  sin(O)=24/25

e.  cos(O)=7/25

f.  tan(O)=24/7

Step-by-step explanation:

Pythagorean Theorem

Note that for the given triangle, one side length is not given, so that will need to be solved for.  Since the triangle is a right triangle, the Pythagorean theorem applies:  \(a^2+b^2=c^2\) where c is the hypotenuse, and "a" and "b" are the two legs.

\(a^2+b^2=c^2\\(7)^2+b^2=(25)^2\\49+b^2=625\\b^2=625-49\\b^2=576\\b= \pm \sqrt{576}\\b= \pm 24\)

Since "b" is a length of a leg of the triangle, we assume it positive and reject the negative solution.

Thus, the missing side is 24 units long.

Trigonometric Definitions

For this problem, remember that trigonometric functions take in an angle, and output a ratio of sides.

So, it is then important to remember which two sides each trigonometric function uses to make its ratio of sides.  Many people remember this with SohCahToa, where the bolded letters represent one of the three trig functions, and the other two letter represent one of the sides of the right triangle

h: hypotenuse (the side across from the right angle)o: opposite (the side across from the angle input to the trig function)a: adjacent (the side that isn't the hypotenuse, touching the angle

So:

       \(\sin(\theta)=\frac{opposite}{hypotenuse}\)       \(\cos(\theta)=\frac{adjacent}{hypotenuse}\)        \(\tan(\theta)=\frac{opposite}{adjacent}\)

So, for each situation, identify which angle is going into the function, then identify which sides you need and where they are.  Substitute and simplify.

Evaluating Trigonometric functions

a.  sin(M)

Note that here the input angle is angle M.  The hypotenuse is always across from the right angle, so the hypotenuse is side MO=25.  The adjacent here is MN=24 (the side touching angle M that isn't the hypotenuse), and the opposite side is ON=7 (the side not touching the angle M)

\(\sin(M)=\frac {opposite}{hypotenuse}\\\sin(M)=\frac {7}{25}\\\)

b.  cos(M)

Again, the input angle is angle M, so the side definitions are the same as in part a.

\(\cos(M)=\frac {adjacent}{hypotenuse}\\\cos(M)=\frac {24}{25}\\\)

c.  tan(M)

Again, the input angle is angle M, so the side definitions are the same as in part a & b.

\(\tan(M)=\frac {opposite}{adjacent}\\\tan(M)=\frac {7}{24}\\\)

d.  sin(O)

Here, the input angle is angle is different from the previous parts of the problem, so we need to look back at our triangle and determine which side is which.

The input angle is angle O.  The hypotenuse is always across from the right angle, so the hypotenuse is side MO=25.  The adjacent here is ON=7 (the side touching angle O that isn't the hypotenuse), and the opposite side is MN=24 (the side not touching the angle O)

\(\sin(O)=\frac {opposite}{hypotenuse}\\\sin(O)=\frac {24}{25}\\\)

e.  cos(M)

Again, the input angle is angle O, so the side definitions are the same as in part d.

\(\cos(O)=\frac {adjacent}{hypotenuse}\\\cos(O)=\frac {7}{25}\\\)

f.  tan(M)

Again, the input angle is angle O, so the side definitions are the same as in part d & e.

\(\tan(O)=\frac {opposite}{adjacent}\\\tan(O)=\frac {24}{7}\\\)

Given g(x) = -1, if g(x) = -16, find x.

Answers

The value of x is -15

What are functions?

Functions are simply defined as expressions or rules showing the relationship between two variables.

These variables are listed as;

The independent variableThe dependent variable

From the information given, we have that;

g(x) =x  -1, if g(x) = -16

Now, we have to equate the two functions, we get;

x - 1 = -16

collect the like terms, we get;

x =-16 + 1

add the values, we have;

x = -15

Learn more about functions at: https://brainly.com/question/11624077

#SPJ1

The complete question:

Given g(x) =x  -1, if g(x) = -16, find x.

As part of a study of the association between smoking and risk of squamous cell carcinoma, a logistic regression model was estimated as:

logit(probability of having squamous cell carcinoma) = -4.84 + 4.6*(SMOKER)

where SMOKER=0 for non-smoker and 1 for smoker. What is the predicted probability of a smoker having squamous cell carcinoma?

Answers

Answer:

0.44

Step-by-step explanation:

Given the estimated logistic regression model on risk of having squamous cell carcinoma

-4.84 + 4.6*(SMOKER)

SMOKER = 0 (non-smoker) ; 1 (SMOKER)

What is the predicted probability of a smoker having squamous cell carcinoma?

exp(-4.84 + 4.6*(SMOKER)) / 1 + exp(-4.84 + 4.6*(SMOKER))

SMOKER = 1

exp(-4.84 + 4.6) / 1 + exp(-4.84 + 4.6)

exp^(-0.24) / (1 + exp^(-0.24))

0.7866278 / 1.7866278

= 0.4402863

= 0.44

A two-piece set of luggage costs 122 If sold individually, the large bag costs 30 more than the small bag. What are the individual costs for each bag?

Answers

Let x be the cost of the small bag.

The large bag costs x + 30 more than the small bag.

So the cost of the large bag is x + (x + 30) = 2x + 30

The two-piece set of luggage cost is the sum of the cost of the small and large bag, so

122 = x + (2x + 30)

Solving this equation for x, we get:

x = 42

Therefore, the small bag costs 42 dollars and the large bag costs 2*42 +30 = 114 dollars.

this is all my 30 points please help
Find the surface area of the composite solid.

this is all my 30 points please helpFind the surface area of the composite solid.

Answers

Answer:

221  I am sorry if I am wrong. I tried

Step-by-step explanation:

There are 328 calories in four ounces of a certain ice cream. How many calories are there in one pound? \text{1 pound}=\text{16 ounces} 1 pound=16 ounces Before you try that problem, answer the question below. How many ounces will you need to find the number of calories for?

Answers

4 ounces ate 328 calories : 1312 calories.

What is unitary method?

This technique allows us to calculate both the value of multiple units from the value of a single unit and the value of single unit from the value of multiple units.

A single unit's value can be determined from the values of multiple units, and multiple units' values can be determined from the values of single units using the unitary technique.

We typically utilize this technique for math calculations.

This approach will be helpful to you while tackling problems involving ratio and proportion, algebra, geometry, etc.

According to our question-

The given 4 ounces ate 328 calories

and 1 pound = 16 ounces

16/4= 4

4 * 328 = 1312

learn more about unitary method click here:

brainly.com/question/24587372

#SPJ1

Provide reasons for the proof of the triangle proportionality theorem. 25 points

Provide reasons for the proof of the triangle proportionality theorem. 25 points
Provide reasons for the proof of the triangle proportionality theorem. 25 points
Provide reasons for the proof of the triangle proportionality theorem. 25 points

Answers

The two column proof is completed below

Statement                                                      Reason

1. line KI parallel to line MN                      Given

2. ∠ JKL ≅ ∠ JMN and ∠ JLK ≅ ∠ JNM   Corresponding angles are equal

3. Δ JKL ~ Δ JMN                                      Similar triangles

4. (JM)/(JK) = (JN)/(JL)                               Ratios of corresponding sides are                            

                                                                  equal

5. JM = JK + KM and JN = JL + LN         Segment Addition Postulate

6. (JK + KM)/(JK) = (JL + LN)/(JL)             Substitution property

7. (JK)/(JK) + (KM)/(JK) = (JL)/(JL) + (LN)/(JL)     Addition of fractions

8. 1 + (KM)/(JK) = 1 + (LN)/(JL)                             Simplify

9. (KM)/(JK) = (LN)/(JL)                                       Simplify (the required proof)

What are similar triangles?

Similar triangles are triangles that have the same shape but not necessarily the same size. In other words, their corresponding angles are equal, and their corresponding sides are proportional.

The equality of corresponding sides of similar triangles is used in the proof and it helped to state that: (JM)/(JK) = (JN)/(JL).

Further calculations were substitution and simplifying

Learn more about Similar triangles at

https://brainly.com/question/14285697

#SPJ1

A diver needs to descend to a depth of 100 feet below sea level. She wants to do it in 5 equal descents. How far should she travel in each descent

Answers

Answer:

She needs to descend 20 feet per descent.

Step-by-step explanation:

100/5 = 20

simplify [Xn (X'nY)]'.​

Answers

Answer:

(X'U'Y) + (XUY')

Step-by-step explanation:

Starting with [Xn(X'nY)]', we can use De Morgan's laws to simplify the expression:

[Xn(X'nY)]' = (Xn)' + (X'nY)'

Recall that Xn represents the logical operator "and", while X' represents "not X". Using these definitions, we can expand the expression:

(Xn)' + (X'nY)' = (X'U'Y) + (XUY')

where U represents the logical operator "or".

Therefore, [Xn(X'nY)]' simplifies to (X'U'Y) + (XUY').

Elsevier logo el Home
Find the area of the combined rectangles.
9 ml
1 2 3 4
The area is
11 ml
19 ml
square miles.
2 ml
8 ml
5
7 ml

Answers

To find the area of the combined rectangles, we need the dimensions (length and width) of each rectangle. However, the provided text and numbers do not seem to correspond to a clear description of the rectangles or their dimensions. Could you please provide more specific information or clarify the question?

Which is the solution for the system of equations? Y = 2x-3
Y = -x
A. (1, -1)
B. No solution
C. (-1, 1)
D. (-1, -1)

Answers

Answer:

(1,-1)

Step-by-step explanation:

\( \large{ \begin{cases} y = 2x - 3 \\ y = - x \end{cases}}\)

Since both are y-isolated equation. We can combine both.

\( \large{2x - 3 = - x}\)

Since we want to solve for x-term, we need to isolate x-term.

So we move from -x which is the right side to x.

\( \large{2x - 3 + x = 0} \\ \large{3x - 3 = 0} \\ \large{3x = 3} \\ \large{x = 1}\)

But we are not done yet. Solving system of equations, you must solve for y-term as well. (Because we have to answer in ordered pairs which are (x, y))

Substitute x = 1 in any given equations but I will substitute x = 1 in y = -x

\( \large{y = - x} \\ \large{y = - 1}\)

Since we have got y-value. Therefore the answer is (1,-1)

Other Questions
Write an explicit formula for an, the nth term of the sequence 33, 30, 27 What is the term for the study of the relationship between individuals and their physical environment which of the following is the most severe potential consequence of failing to comply with regulations for the handling of trust funds? Please please help help ASAP Drag each tile to the correct box.Match each sentence from the passage with the statement that explains how it develops the authors thesis.Attachment Below 2) Evaluate the integral and check your answer by differentiating. -2x3 dx a) a) 1'" Choose all of the terms that refer to World War I. A) D-Day B) Axis Powers trench warfare D) Treaty of Versailles E The Zimmerman Telegram PLEASE HELP IM GONNA GET IN TROUBLE what the slope and the y intercept for-12x+4y=-8 Which expression is equivalent to1216 the advantage of the myelin sheath in vertebrates is that it prevents across the membrane, resulting in an increase in . in a myelinated axon nearly all of the ion channels are concentrated in the ; this organization allows a special kind of impulse propagation called . What causes some hurricanes to be more damaging than others? Explain. Will the square of a negative number always be positive? Explain why.Will the cube of a negative number always be negative? Explain why. consider the market for ipods. what happens if a fantastic new alternative mp3 player is developed and, at the same time, a boat carrying a large shipment of ipods is attacked by sea monsters and sunk? a price decreases and quantity increases. b price decreases and the change in quantity is uncertain. c the change in price is uncertain and quantity decreases. d price increases and quantity increases. e price increases and the change in quantity is uncertain. Dendrochronology is a method of dating by usingtree rings.radiocarbon.calendar age.correlated age. Use the excerpt from the petition to Parliament to answer the question.The author of the petition would MOST LIKELY agree with what reason for resisting British mercantilist policies?A. British laws are too protectionist and hurt the colonies ability to provide for themselves.B. British laws undermine colonial currency by inflating the costs of British manufactured goods.C. British laws promote consumerism in order to secure more profits for British companies.D. British laws require too much labor and dont leave the colonists time for their own work. 5(28m)=10+5mShow step by step Social studies help!!! Please dont guess for points! Its annoying! Why is the western part of Washington State more likely to have a higher population? It has an overabundance of oil deposits. It has milder climate and access to the sea. It receives very little rainfall. It has mountains that contain great farmland. a rectangle has an area of 240 square inches and a length which is six inches more than three times the width. find the length and the width of the rectangle. A pizza parlor is creating a questionnaire to survey customers about their satisfaction with the taste of their pizzas. Which questions would be considered good questions to include on the survey?